LSAT and Law School Admissions Forum

Get expert LSAT preparation and law school admissions advice from PowerScore Test Preparation.

 Administrator
PowerScore Staff
  • PowerScore Staff
  • Posts: 8916
  • Joined: Feb 02, 2011
|
#23626
Complete Question Explanation

Strengthen—Principle. The correct answer choice is (B)

The argument in the stimulus is formulated as follows:

     Premise: Some scientists have expressed reservations about quantum theory because of
     its counterintuitive consequences.

     Premise: But despite rigorous attempts to show that quantum theory’s predictions were
     inaccurate, they were shown to be accurate within the generally accepted
     statistical margin of error.

     Conclusion: These results, which have not been equaled by quantum theory’s competitors,
     warrant acceptance of quantum theory.

The argument uses the “some people say” construction. As some scientists
are said to have reservations about quantum theory, you can predict that the author will conclude that
they should not have these reservations, and indeed this occurs in the last sentence of the stimulus.

The author’s reason for concluding that quantum theory should be accepted is that all attempts to
disprove the theory have failed. From the Flaw in the Reasoning section we know this is an error in
the use of evidence: lack of evidence against a position is mistakenly taken to prove that position
is true. The question stem asks you to identify a principle that could help to justify this form of
erroneous reasoning.

Answer choice (A): The author’s reasoning is not based on “fewer counterintuitive consequences,”
but on the fact that the consequences have not been disproven. Hence, this principle would not help
the reasoning and this answer is incorrect.

Answer choice (B): This is the correct answer choice. If this principle is accepted, then the argument
in the stimulus is strengthened. The principle, when slightly restated, asserts that “if a scientific
theory has been subjected to serious attempts to disprove it and has withstood them, then it should
be accepted.” We know from the stimulus that the theory has been subject to serious attempts to
disprove it and it has withstood them. According to the principle, this justifies the conclusion that the
theory should be accepted.

Answer choice (C): The conclusion of the argument in the stimulus is that the theory should be
accepted. Using the principle in this answer choice would lead one to conclude that the consequences
of the theory should not be considered counterintuitive. Since this result differs from the conclusion,
this answer choice is incorrect.

Answer choice (D): This answer, when rephrased using the Unless Equation, reads “if a theory
should be rejected, then it has been subjected to serious attempts to disprove it.” The scenario in the
stimulus meets the necessary condition, but that has no impact on whether the sufficient occurs or
does not occur.

Answer choice (E): This is the Mistaken Reversal of answer choice (B). Applying this principle
fails to justify the argument since only the necessary condition is met, and that does not prove the
sufficient condition (in this case, that the theory should be accepted) will occur. Hence, this answer
choice is incorrect.
 Sdaoud17
  • Posts: 85
  • Joined: Apr 13, 2013
|
#9527
Can you provide me with the explanation of A, C , D and E why they are also wrong?

Thank you
 Lucas Moreau
PowerScore Staff
  • PowerScore Staff
  • Posts: 216
  • Joined: Dec 13, 2012
|
#9538
A is incorrect because the reasoning says that, despite quantum theory's counterintuitive consequences, it should be accepted because its predictions were shown to be inaccurate. The number of counterintuitive consequences is not at issue, nor do we know if quantum theory actually does have less counterintuitive consequences than its competitors.

C is incorrect because quantum theory's counterintuitive consequences continue to be considered counterintuitive, even after its predictions are shown to be accurate. There is nothing to indicate that quantum theory's counterintuitive consequences have been shown to be any less counterintuitive as a result of its predictions being shown to be accurate.

D is incorrect because this principle is simply not stated. It is not that quantum theory should not be rejected because it had not been subject to attempts to disprove it, it is that quantum theory should not be rejected because the attempts to disprove it have failed. The mere fact of there being serious attempts to disprove it does not affect whether or not a theory should be rejected.

E is incorrect because quantum theory's predictions have not been proven by experiment, they have merely failed to be disproven. And moreover, this fact is not the only reason quantum theory should be accepted, or at least, that is not demonstrated by the stimulus.

I hope that helps!
 kristinaroz93
  • Posts: 160
  • Joined: Jul 09, 2015
|
#19342
"Some scientists have expressed reservations about quantum theory because of its counterinuitive consequences..."

I get to a point where I get questions right, but I cannot always explain why, which really frustrates me. This is one of those questions. Can we go over the diagramming of each answer choice individually and why it is right or wrong or in other words how it relates or does not relate back to the stimulus. And of course the diagramming of the stimulus itself!

And also: does not disproved= accurate
and not rejected = accepted.

If the above is correct, then I elminated E for being a mistaken reversal.
stimulus: accurate-->acceptance
E: accepted--> not disproved/accurate

Maybe there isn't even conditional reasoning in ths question, but the online explanation was so brief I could not decipher too much. Which is why I would like to go through each choice now and understand why B is right and the others are wrong.

Thanks in advance!
 David Boyle
PowerScore Staff
  • PowerScore Staff
  • Posts: 836
  • Joined: Jun 07, 2013
|
#19352
kristinaroz93 wrote:"Some scientists have expressed reservations about quantum theory because of its counterinuitive consequences..."

I get to a point where I get questions right, but I cannot always explain why, which really frustrates me. This is one of those questions. Can we go over the diagramming of each answer choice individually and why it is right or wrong or in other words how it relates or does not relate back to the stimulus. And of course the diagramming of the stimulus itself!

And also: does not disproved= accurate
and not rejected = accepted.

If the above is correct, then I elminated E for being a mistaken reversal.
stimulus: accurate-->acceptance
E: accepted--> not disproved/accurate

Maybe there isn't even conditional reasoning in ths question, but the online explanation was so brief I could not decipher too much. Which is why I would like to go through each choice now and understand why B is right and the others are wrong.

Thanks in advance!
Hello,

"Not disproved" should not mean "accurate", I don't think. Nor should "not rejected" always mean "accepted", either. (Maybe something needs further study, even if it hasn't been rejected)

Answers:

A: fewer counterintuitive consequences :arrow: accepted
B: withstood all of serious attempts to disprove :arrow: accepted
C: accurate predictions :arrow: consequences not counterintuitive
D: rejection :arrow: serious attempts to disprove
E: accepted :arrow: predictions not experimentally disproved

Answer A doesn't really work, since we're not comparing number of counterintuitive consequences with other theories. C doesn't work, since we're not claiming that accuracy will cure the problem of counterintuitive consequences. D is wrong, since there may be other reasons to reject a theory besides serious attempts to disprove it.

E sort of resembles a mistaken reversal of B, though isn't one, exactly. For one, the stimulus doesn't mention "experiments" at all; conversely, E doesn't mention "rigor" at all.
And B, obviously, is fine, since it chimes with the stimulus saying that quantum theory withstood some big challenges.

David
 kristinaroz93
  • Posts: 160
  • Joined: Jul 09, 2015
|
#19372
Hi David,

Thanks for your in depth explanation! I have a few questions to add:

Can we eliminate d: Due to the idea that ~rejected does not equal accepted. And we are trying to infer what must happen for something to be accepted?
In fact can we just reject D for talking about rejection at all, since rejection is not talked about in the stimulus?

and for E what do you mean by rigor? (would it be enough to elminate E on the term experiment a lone ?)
 Herzog.Laura
PowerScore Staff
  • PowerScore Staff
  • Posts: 7
  • Joined: Jun 30, 2015
|
#19380
Hi Kristina,

I think that is a good way to look at answer choice (D). Knowing the conditions necessary to reject a theory, doesn't help us to decide whether or not to accept it, considering that we have established that not accept doesn't equal reject, and that not reject doesn't equal accept.

Regarding answer choice (E), I think that you could reject it based on "experiments" alone. However, the way I understand what David was saying about "rigor" is that the stimulus specifically refers to the quality of the experiments (that they were rigorous). The answer choice doesn't tell us anything about how/ to what standards the experiments were performed.
 kristinaroz93
  • Posts: 160
  • Joined: Jul 09, 2015
|
#20258
I have a question regarding problem 4 : "Some scientists have expressed reservations about quantum theory because of its counterintuitive consequences..."

I do not quite understand the conditional reasoning behind this problem and how it pertains to each answer choice. Would someone be so kind as to diagram out the stimulus and then compare it against all the choices to make the problem more clear=)

Thanks in advance!
User avatar
 Dave Killoran
PowerScore Staff
  • PowerScore Staff
  • Posts: 5853
  • Joined: Mar 25, 2011
|
#20259
Hi Kristina,

Thanks for the question! This problem is likely giving you difficulty in part because of the topic, and all of the scientific jargon they toss around in the stimulus. At a quick glance, this question looks like it will be hard.

You mention diagramming this problem, and while there is a breakdown of the argument structure in the explanation, there is no accompanying diagram. This is because I wouldn't diagram this question :-D You and I have discussed this topic before, but the majority of problems on the LSAT shouldn't or can't be diagrammed. When you start out a problem, you shouldn't think about diagramming it; you should think about what's being said and how the argument shapes up in terms of premises and conclusions. Then, if you see an element that absolutely compels you to diagram, that's when you should do it. In a nutshell, don't look at these problems and try to find a way to diagram them; instead, wait until the problem forces you to diagram it. Does that makes sense? It's a small difference in approach, but it has big consequences.

Ok, back to the problem. The thing we want to establish here is, what the heck are they saying? I'd summarize the stimulus in the following manner:

  • We've got some scientists who aren't sure that quantum theory is really legit. But people have tried to disprove the theory before, and it always works and comes out with good predictions. So, because there is nothing better, we should all accept quantum theory.
The last sentence is the conclusion, and I've put that in bold for emphasis. What it really comes down to is that some people are suspicious of the theory, but it seems to work and nothing else is better, so the author thinks we should just accept it.

From there, you are asked to Strengthen the argument with a principle. Thus, we need an answer that supports the conclusion that we should accept the theory, and hopefully relate that acceptance to one or more of the premises that state that the theory gives good predictions and nothing else seems to be better.

Ok, with all of that in place and assuming that what I've said so far makes sense, here's what I want you to do: break down each answer for me. Don't worry about logical terms or fallacy names or anything like that. Just tell me what's right/wrong with each problem in the briefest terms possible—somewhat similar to how you'd do it on the test. Is that ok?

Thanks!
 kristinaroz93
  • Posts: 160
  • Joined: Jul 09, 2015
|
#20534
Hi Dave,

Thanks for your response!

After coming back to this problem after redoing the entire bible for a second time around, here is how I aproached it. I think the conditional is in the answer choices not in the stimulus itself having come back to the problem since it is a strenghthen/justify principle, and we need a choice that helps the conclusion in the stimulus be drawn. I find this problem similar to the one on page 600 of the bible!

Diagram for principal that helps prove conclusion in stimulus: Theory not disproven/shown to be accurate within the generally accepted statistcical margin of error --->theory warrants acceptance.

For B, it matches exactly what I wrote above. And it allows the conlcusion of the stimulus to be drawn, since we have the attempts to disprove quantum theory and consequent failure of dissaproval stated in the stimulus, which means we can draw the conclusion that quatum theory theory warrants acceptance.

E, on the other hand, puts "predictions not disproved" on the neccessary side which means we can't conclude anything from this further if we only know that the necessary is met looking back at the stimulus! And so this answer is wrong. And like stated in the bible, it is just a mistaken reversal of B.

D is problematic since, like E, we can't conclude anything from a necessary condition. I also think it is problematic because it says "subjected to serious attempts to disprove it", but doesn't tell us if it has actually been disproven or not. And we also do not know when a theory warrants rejection, we only know when it warrants acceptance.

And again A and C talk about counterintuitive consequences, which is just a smokescreen of the problem (i.e. meant to make the reading longer) I think.

Please let me know your feedback for my reasoning=)

Get the most out of your LSAT Prep Plus subscription.

Analyze and track your performance with our Testing and Analytics Package.